Difference between revisions of "2018 AMC 8 Problems/Problem 14"
(→Solution(factorial)) |
(→Solution 2 (Factorial)) |
||
(7 intermediate revisions by 2 users not shown) | |||
Line 4: | Line 4: | ||
<math>\textbf{(A) }15\qquad\textbf{(B) }16\qquad\textbf{(C) }17\qquad\textbf{(D) }18\qquad\textbf{(E) }20</math> | <math>\textbf{(A) }15\qquad\textbf{(B) }16\qquad\textbf{(C) }17\qquad\textbf{(D) }18\qquad\textbf{(E) }20</math> | ||
− | == Solution == | + | == Solution 1== |
If we start off with the first digit, we know that it can't be <math>9</math> since <math>9</math> is not a factor of <math>120</math>. We go down to the digit <math>8</math>, which does work since it is a factor of <math>120</math>. Now, we have to know what digits will take up the remaining four spots. To find this result, just divide <math>\frac{120}{8}=15</math>. The next place can be <math>5</math>, as it is the largest factor, aside from <math>15</math>. Consequently, our next three values will be <math>3,1</math> and <math>1</math> if we use the same logic. Therefore, our five-digit number is <math>85311</math>, so the sum is <math>8+5+3+1+1=18\implies \boxed{\textbf{(D) }18}</math>. | If we start off with the first digit, we know that it can't be <math>9</math> since <math>9</math> is not a factor of <math>120</math>. We go down to the digit <math>8</math>, which does work since it is a factor of <math>120</math>. Now, we have to know what digits will take up the remaining four spots. To find this result, just divide <math>\frac{120}{8}=15</math>. The next place can be <math>5</math>, as it is the largest factor, aside from <math>15</math>. Consequently, our next three values will be <math>3,1</math> and <math>1</math> if we use the same logic. Therefore, our five-digit number is <math>85311</math>, so the sum is <math>8+5+3+1+1=18\implies \boxed{\textbf{(D) }18}</math>. | ||
+ | == Solution 2 (Factorial) == | ||
+ | <math>120</math> is <math>5!</math>, so we have <math>(5)(4)(3)(2)(1) = 120</math>. (Alternatively, you could identify the prime factors <math>(5)(3)(2)(2)(2) = 120</math>.) Now look for the largest digit you can create by combining these factors. | ||
− | = | + | <math>8=4 \cdot 2</math> |
− | + | Use this largest digit for the ten-thousands place: <math>8</math>_ , _ _ _ | |
− | <math> | + | Next you use the <math>5</math> and the <math>3</math> for the next places: <math>85,3</math> _ _ (You can't use <math>3 \cdot 2=6</math> because the <math>2</math> was used to make <math>8</math>.) |
− | + | ||
+ | Fill the remaining places with 1: <math>85,311</math> | ||
− | |||
<math> = (5)(8)(3)(1)(1) =120</math> | <math> = (5)(8)(3)(1)(1) =120</math> | ||
− | + | <math>8+5+3+1+1=\boxed{\textbf{(D) }18}</math>. | |
− | |||
− | |||
− | |||
− | |||
− | <math>8+5+3+1+1=18</math> | ||
==Video Solution (CREATIVE ANALYSIS!!!)== | ==Video Solution (CREATIVE ANALYSIS!!!)== |
Latest revision as of 09:01, 10 April 2024
Contents
Problem
Let be the greatest five-digit number whose digits have a product of . What is the sum of the digits of ?
Solution 1
If we start off with the first digit, we know that it can't be since is not a factor of . We go down to the digit , which does work since it is a factor of . Now, we have to know what digits will take up the remaining four spots. To find this result, just divide . The next place can be , as it is the largest factor, aside from . Consequently, our next three values will be and if we use the same logic. Therefore, our five-digit number is , so the sum is .
Solution 2 (Factorial)
is , so we have . (Alternatively, you could identify the prime factors .) Now look for the largest digit you can create by combining these factors.
Use this largest digit for the ten-thousands place: _ , _ _ _
Next you use the and the for the next places: _ _ (You can't use because the was used to make .)
Fill the remaining places with 1:
.
Video Solution (CREATIVE ANALYSIS!!!)
~Education, the Study of Everything
Video Solutions
https://youtu.be/7an5wU9Q5hk?t=13
~savannahsolver
See Also
2018 AMC 8 (Problems • Answer Key • Resources) | ||
Preceded by Problem 13 |
Followed by Problem 15 | |
1 • 2 • 3 • 4 • 5 • 6 • 7 • 8 • 9 • 10 • 11 • 12 • 13 • 14 • 15 • 16 • 17 • 18 • 19 • 20 • 21 • 22 • 23 • 24 • 25 | ||
All AJHSME/AMC 8 Problems and Solutions |
The problems on this page are copyrighted by the Mathematical Association of America's American Mathematics Competitions.